Difference between revisions of "1965 AHSME Problems/Problem 3"
m (→Added (B) choice) |
Megaboy6679 (talk | contribs) m |
||
(6 intermediate revisions by 3 users not shown) | |||
Line 7: | Line 7: | ||
== Solution == | == Solution == | ||
− | + | Let us recall <math>\text{PEMDAS}</math>. We calculate the exponent first. <math>(-2)^{-2}=\frac{1}{(-2)^2}=\frac{1}{4}</math> When we substitute, we get <math>81^{\frac{1}{4}}=\sqrt[4]{81}=\boxed{\textbf{(C) }3}</math>. | |
+ | |||
+ | ~Mathfun1000 | ||
+ | |||
+ | ==See Also== | ||
+ | {{AHSME box|year=1965|num-b=2|num-a=4}} | ||
+ | {{MAA Notice}} | ||
+ | [[Category:AHSME]][[Category:AHSME Problems]] |
Latest revision as of 20:15, 10 January 2023
Problem
The expression has the same value as:
Solution
Let us recall . We calculate the exponent first. When we substitute, we get .
~Mathfun1000
See Also
1965 AHSME (Problems • Answer Key • Resources) | ||
Preceded by Problem 2 |
Followed by Problem 4 | |
1 • 2 • 3 • 4 • 5 • 6 • 7 • 8 • 9 • 10 • 11 • 12 • 13 • 14 • 15 • 16 • 17 • 18 • 19 • 20 • 21 • 22 • 23 • 24 • 25 • 26 • 27 • 28 • 29 • 30 | ||
All AHSME Problems and Solutions |
The problems on this page are copyrighted by the Mathematical Association of America's American Mathematics Competitions.